LSAT and Law School Admissions Forum

Get expert LSAT preparation and law school admissions advice from PowerScore Test Preparation.

 EL16
  • Posts: 45
  • Joined: Jul 06, 2017
|
#38994
Hi,

I am very confused between some inconsistencies I think I am finding between the Administrator's initial post above, and some later posts here explaining some of the diagramming. The Administrator's initial post says that part of the stimulus should be diagrammed as follows:

If McConnell runs, that must mean that she has found something “dirty” in the record.
  • M RUN ..... :arrow: ..... Lutz Dirty
However, later PS instructor posts seem to indicate the opposite, and that this sentence of the stimulus should actually be diagrammed as:

L Dirty/scandal :arrow: McConnell runs

In my personal diagramming, I diagrammed it the same as the 2nd option above, but now I am so confused with this since it seems to be a mistaken reversal of the administrator's diagramming? Could someone please help me to understand which way is correct? This seems to make a big difference in understanding answer C versus answer D.

Thanks,
Elana
 nicholaspavic
PowerScore Staff
  • PowerScore Staff
  • Posts: 271
  • Joined: Jun 12, 2017
|
#39465
Hi Elana,

Good catch! :-D The diagramming of "If McConnell runs, that must mean that she has found something “dirty” in the record" is:

M RUN ..... :arrow: ..... Lutz Dirty

However that's not what the stimulus says. It reads: "Anything of a scandalous nature would increase McConnell's chances of winning, and she would campaign for election." So to be clear the correct diagramming of the stimulus as written is actually:

McConnell finds hints of scandal :arrow: McConnell runs

And you are also correct about the Mistaken Reversal in the Admin post. So well done!

We are going to clean up the Admin's post but it's clear that you are definitely developing the LSAT eye, so well done all around!
 EL16
  • Posts: 45
  • Joined: Jul 06, 2017
|
#39603
Thank you for clearing that up, Nicholas!
 Mark83
  • Posts: 32
  • Joined: Sep 22, 2017
|
#47957
Lucas Moreau wrote:Hello, Charles,

This one is tricky - and so early on, too. :ras: The key is to diagram this question out into branches. The first branch is whether or not Lutz has a large campaign fund.

Lutz has a large campaign fund :arrow: McConnell does not run
Lutz does not have a large campaign fund :arrow: McConnell scrutinizes Lutz's record

Let's call the first outcome A, and the second one B. Now if we get to this second part B, McConnell scrutinizing Lutz's record, then it branches again.

McConnell finds hints of scandal :arrow: McConnell runs
McConnell does not find hints of scandal :arrow: McConnell does not run

Let's call that first outcome B1, and the second one B2. So, to recap:

A: McConnell does not run
B1: McConnell runs
B2: McConnell does not run

Answer choice C is not guaranteed to be false. If Lutz's record contains scandalous items, McConnell might very well find them, which would lead to outcome B1, McConnell running against Lutz. McConnell might not find those items, but McConnell seems tenacious to me. :-D

Answer choice D, though, doesn't allow for any of the outcomes. If Lutz's record contains no hints of scandal, then McConnell cannot possibly find any, and so we are forced into outcome B2, McConnell does not run. But since the answer choice says Lutz's record is clean and McConnell runs, that is outside the rules of the question, and so it cannot be true.

Hope that helps,
Lucas Moreau
Is the reason (C) could be true because even if Lutz record contains scandalous items, McConnell still might not run if Lutz has a larger campaign fund? That is to say, only one of the conditions for McConnell not running has been satisfied.

Whereas in (D) the condition has not been satisfied for Mcconell not to run but she's running anyway?
 Adam Tyson
PowerScore Staff
  • PowerScore Staff
  • Posts: 5153
  • Joined: Apr 14, 2011
|
#48199
Your analysis of answer C is perfect, Mark83!

I would put it a little differently for answer D, although we might be saying the same thing here. In answer D, a sufficient condition for her NOT running (Lutz has a clean record) has been met, and she runs anyway, failing to meet the necessary condition.

Good work!

Get the most out of your LSAT Prep Plus subscription.

Analyze and track your performance with our Testing and Analytics Package.